Download Chapter 1

Survey
yes no Was this document useful for you?
   Thank you for your participation!

* Your assessment is very important for improving the workof artificial intelligence, which forms the content of this project

Document related concepts

Kidney transplantation wikipedia , lookup

Urinary tract infection wikipedia , lookup

Kidney stone disease wikipedia , lookup

Transcript
Chapter 1
1. Voluntary micturition is initiated by inhibition of the:
A. Parasympathetic nerves.
B. Hypogastric sympathetic nerves.
*C. Pudendal nerve.
D. Facial nerves.
E. Glossopharyngeal nerves.
2. For which compound the excretion can be higher than the filtered load?
A. Glucose.
B. Inulin.
C. Creatinine.
*D. PAH.
E. None is correct.
3. A freely filterable substance that neither reabsorbed nor secreted has a
renal artery concentration of 12 mg/ml and a renal vein concentration of 9
mg/ml. Which of the following is the filtration fraction (GFR/RPF)?
A. 0.05.
B. 0.15.
*C. 0.25.
D. 0.35.
E. 0.45.
4. Assuming the GFR =100 ml/min and Tm = 375 mg/min, indicate the
glucose threshold plasma concentration (mg/ml) at which the glucose will
appear in urine:
A. 8.75.
B. 5.75.
*C. 3.75
D. 2.75
E. 1.75.
5. Laboratory tests have shown that the renal clearance of certain compound
is 50 ml/min. Indicate the specific mechanism of renal excretion.
A. Filtration.
*B. Filtration + Reabsorption.
C. Secretion + Reabsorption.
D. Secretion + Filtration.
E. None is correct.
6. If a substance appears in the renal artery but not in the renal vein,
A. Its clearance is equal to GFR.
B. It must be reabsorbed by the kidney.
C. Its urinary concentration must be higher than its plasma concentration.
*D. Its plasma clearance is equal to RPF.
E. It has the “filtration only” mechanism.
7. A previously well 12-year-old boy is brought to the emergency room with
vomiting after a prolonged period of exercise. Laboratory findings (high serum
creatinine and urea) suggest acute renal failure. Following treatment and
recovery, his serum uric acid concentration (0.6 mg/dL) remains consistently
below normal. What is the uric acid clearance if V = 1ml/min and Uuric acid =
36 mg/dL.
A. 6 ml/min.
B. 12 ml/min.
C. 24 ml/min.
D. 48 ml/min.
*E. 60 ml/min.
8. Use the following laboratory data to estimate the renal blood flow
(RPF) by assuming that PAH (p-aminohipuric acid) does not returns to
systemic circulation:
Renal artery PAH = 0.15 mg/ml
Urinary PAH = 19 mg/ml.
Urine flow = 5 ml/min.
RPF is approximately:
A. 800 ml/min.
B. 733 ml/min
*C. 633 ml/min.
D. 590 ml/min.
E. 520 ml/min.
Chapter 2.
1. A fall of PGC from 59 mm Hg to 49 mm Hg at afferent arteriole in
dehydrated individual will cause.
*A. A drop of GFR to less then half.
B. A drop of GFR by 20%.
C. No change of creatinine in blood.
D. Two fold decrease of creatinine in blood.
E. No change in RBF.
2. Mark the incorrect answer. Vasodilation of the efferent arteriole will
cause the following effects:
A. GFR will decrease little bit.
B. RBF will increase.
*C. Filtration fraction will stay constant.
D. Filtration fraction will decrease.
E. RPF will increase.
3. GFR and RBF will both be increased if:
*A. Only the afferent arterioles is dilated.
B. The efferent and afferent arterioles are both constricted.
C. Only the afferent arteriole is constricted.
D. Only the efferent arteriole is constricted.
E. The afferent arteriole is constricted and the efferent is dilated.
4. Under certain patho-physiological conditions (e.g. hemorrhage) the
decreased levels of RBF may cause kidney ischemia. The release of which
hormone may prevent the kidney ischemia?
A. ADH.
B. ANP.
C. Renin.
D. Aldosterone.
*E. PGI2 and PGE2.
5. Mark the incorrect answer. Macula Densa responds to the increased
NaCl delivery into the tubule by:
A. Releasing adenosine.
B. Constriction of the afferent arteriole of the same nephron.
C. Decreasing the GFR back to normal value.
D. Decreasing the RBF back to normal value.
*E. Increasing both the GFR and RBF back to normal value.
6. A release of which hormone will cause a modest increase in GFR with
little change in RBF?
A. Adenosine.
*B. ANP.
C. AngiotensineII.
D. Epinephrine.
E. Endotheline.
7. Mark the incorrect answer. A prolonged physical exercise of healthy
person results in increase of arterial blood pressure from 120 mm Hg to 170
mm Hg. This might be associated with:
A. Vasoconstriction of afferent arterioles.
B. No significant change in filtered load of Na+.
C. Substantial increase in Na+ excretion.
*D. No change in Na+ reabsorption.
E. No significant change in GFR and RBF.
8. The filtration fraction is increased by which of the following?
A. Increasing renal blood flow.
B. Increasing afferent arteriolar resistance.
*C. Increasing efferent arteriolar resistance.
D. Increasing plasma oncotic pressure.
E. Increasing the pressure within the Bowman’s capsule.
------------------------------------------------------------------------------------------------------Chapter 3.
1. A 39-yer-old women is admitted to the hospital by her physician
because of decreased urine output. To distinguish the volume depletion from
renal failure the sodium excretion fraction must be estimated. The laboratory
tests reveal:
Urine:
Serum:
PNa = 135 mEq/L
UNa= 40 mEq/L
PCr = 70 mg/L
UCr = 1000 mg/L
So, her fractional sodium excretion is approximately:
A. 0.5%.
B. 1.0%.
C. 1.5%.
*D. 2.0%.
E. 3.0%.
2. Mark the incorrect answer. Inhibition of the following systems may cause
a diuretic effect.
A. Loop Na+/K+/Cl- contransporter.
B. Distal tubule Na+/Cl- contransporter.
C. Proxmal tubular carbon anhydrase.
*D. Collecting duct Ca2+-ATPase.
E. Collecting duct Na+-channel.
3. If GFR increases, proxymal tubular reabsorption of salt and water will
increase by a process called GT-balance. Contributions to this process
includes which of the following?
A. An increase in peritubular capillary hydrostatic pressure.
B. A decrease in peritubular capillary sodium concentration.
*C. An increase in peritubulary capillary oncotin pressure.
D. An increase of urine flow in proximal tubule.
E. An increase of urine flow in collecting duct.
4. What percentage of the filtered load of sodium is reabsorbed by the
proximal tubule?
A. 20%
B. 25%.
C. 45%.
*D. 65%.
E. 95%.
5. Mark the incorrect answer. Sodium reabsorption is coupled with:
A. Proton secretion.
B. Potassium secretion.
C. Bicarbonate reabsorption.
*D. Amino acids secretion.
E. Glucose reabsorption.
6. Potassium-sparing diuretics inhibit sodium reabsorption in which of the
following:
A. Proximal tubule.
B. The descending limb of Henle’s loop.
C. The ascending limb of Henle’s loop.
*D. Collecting duct.
E. None is correct.
7. Na+ is reabsorbed from the basolateral surface into the intersticium by
which of the following:
A. Na/H exchange.
B. Na/glucose cotransport.
*C. Na/K pump.
D. Na/amino acid cotransport.
E. Carbon anhydrase.
Chapter 4.
1. Mark the incorrect answer. A rapid i.v. infusion of 1L isotonic NaCl
solution to healthy person will result in:
*A. 15-20% increase of Posm.
B. Increase of blood volume.
C. Increase of ECV.
D. Decrease of ADH levels in plasma.
E. Acceleration of urine formation.
2. If a patient with diabetes Insipidus exhibits ClH2O = 0.5 L/hr, how much
water must that patient drink each hour to maintain normal osmolality of body
fluids?
A. 2 L/day.
B. 5 L/day.
*C. 0. 5 L/hr.
D. 1 L/hr.
E. 2 L/hr.
3. Mark the incorrect answer. A production of hyperosmotic urine is
associated with:
A. urine formation (V).
*B.  UosmV (total solute excretion).
C. Negligible changes in Posm (plasma osmolality).
D. No significant changes in Closm (total solute clearance).
E.  TCH20 (tubular conservation of water).
4. Mark the incorrect answer. The following is required for production
of hyperosmotic urine:
A. A renal response to ADH.
B. A 150-200 mOsm/L difference at the entry and exit of Henle’s loop.
C. An osmolality gradient (300-1200 mOsm/L) in the medulla.
*D. An active production of ‘single effect’ by vasa recta.
E. A countercurrent exchange in Henle’s loop of jugstaglomerular
nephron.
5. Constriction of a renal artery (renal artery stenosis) in one kidney (by
an atherosclerotic plaque, for example) may result in:
A. Elevated renin secretion from both kidneys.
B. Reduced renin secretion from both kidneys.
C. Decreased arterial blood pressure in healthy kidney.
D. Elevated sympathetic stimulation of both kidneys.
*E. Reduced renin secretion from healthy kidney.
6. A 28-year old woman says she is always thirsty and has to empty the
bladder very frequently. Laboratory tests show Posm= 305 mOsm/L, Uosm =
90 mOsm/L and elevated plasma levels of ADH. Her urine contains no
glucose. The most likely diagnosis is:
A. Central diabetes insidipus.
B. Diabetes mellitus.
*C. Nephrogenic diabetes insidipus.
D. Primary polydipsia.
E. Abuse of diuretics.
7. If the observed value of free water clearance is ClH2O = -1L/day that
means that the kidneys:
A. Excrete 1 L/day of free water.
*B. Add 1 L/day of free water to ECF.
C. Produce 1L/day of urine.
E. Produce isotonic urine.
F. Produce hypotonic urine.
8. The syndrome of inappropriate ADH hormone secretion (SIADH) is
caused by the excess release of ADH. SIADH will cause an increase in which
of the following?
A. Concentration of plasma sodium.
*B. ICF.
C. Urinary flow.
D. Plasma oncotic pressure.
E. Plasma osmolality.
------------------------------------------------------------------------------------------------------Chapter 5.
1. A concurrent intake of Spironolactone (aldosterone antagonist) and
ACE (Angiontensin Converting Enzyme) inhibitor is dangerous because they
may cause:
A.
*B.
C.
D.
E.
Hypernatremia.
Hyperkalemia.
Elevated Na+-reabsorption.
Elevated K+-secretion.
Elevated K+-excretion.
2. Mark the incorrect answer. Proteinurea may occur in following cases:
A. Loss of negative charge on the filtration barrier.
B. Breakdown of blood red cells (hemolysis).
C. Breakdown of muscle cells (rhabdomyolysis).
*D. Nephrogenic diabetic insidipus.
E. Impaired hydrolysis of proteins in proximal tubule.
3. Mark the incorrect answer. Hypoaldosteronism (low levels of
aldosterone in plasma) may cause the following effects:
A. Low tubular reabsorption of Na+.
B. Acidosis + Hyperkalemia.
*C. Pulmonary edema.
D. Reduced ECV.
E. Reduced H+-secretion in the collecting duct.
4. The mechanism of glomerulotubular balance (GT-balance) is
responsible for retaining a constant:
*A. Filtration fraction.
B. Na+ excretion (mEq/day).
C. Free water clearance (ClH2O).
D. Plasma osmolality (Posm).
E. Urine osmolality (Uosm).
5. Decreased renal sympathetic activity will cause the following effects:
Na+ + H2O
GFR
ADH
Renin
Aldosterone
Excretion
A





B





C





D





*E





6. A 23-year-old man is brought to the emergency room after collapsing
during basketball practice. On admission he appears confused. His coach
reports that he was drinking a lot of water during practice. His symptoms are
most likely caused by increased
A. Intracellular tonicity.
B. Exracellular tonicity.
C. Hypernatremia.
*D. Intracellular volume.
E. Hyperkanemia.
7. Renin release from the juxtaglomerular apparatus is inhibited by
which of the following:
A. Beta-adrenergic agonists.
B. Prostaglandins.
C. Aldosterone.
D. Stimulation of Macula Densa.
*E. Increased hydrostatic pressure within the afferent arterioles.
8. An individual is euvolemic and ingests a diet that contains
200 mEq /day of sodium on average. What would be the sodium excretion
rate of this individual over a 24-hour period?
A. 400 mEq/day.
*B. 200 mEq/day.
C. 120 mEq/day.
D. 100 mEq/day.
E. 50 mEq/day.
Chapter 6.
1. The amount of potassium excreted by the kidney will decrease if
A. Distal tubular flow increases.
B. Circulating aldosterone levels increase.
C. Dietary intake of potassium increases.
*D. Na+-reabsorption by the collecting duct decreases.
E. The excretion of organic ions decrease.
2. The extracellular potassium of a hyperkalemic patient can be decreased
by administering which of the following
*A. Epinephrine.
B. Lactic acid.
C. Isotonic saline.
D. Hypertonic saline.
E. Loop diuretics.
3. A rapid reabsorption of K+ by gastrointestinal system (e.g., after a
meat meal) may cause dangerous levels of hyperkalemia. This can be
prevented by immediate
A. Release of ADH in blood.
B. Release of Aldosterone in blood.
*C. Release of Insulin in blood.
D. Elevation in H+-secretion.
E. Elevation in K+-secretion.
4. Which hormone inhibits the renal excretion of both the phosphate and
calcium:
A. PTH.
B. Calcitriol.
*C. Calcitonin.
D ADH.
E. Angiotensin II.
5. Hyperkalemia may be observed in patients with which of the following?
A. Volume depletion.
B. Diuretic therapy.
C. Administration of insulin.
*D. Metabolic alkalosis.
E. ADH release.
6. Mark the incorrect answer. Parathyroid hormone (PTH) increases the
plasma calcium by
A. Stimulating bone resorption.
B. Increasing the calcium reabsorption by the kidney.
C. Stimulating the production of calcitriol.
D. Increasing (indirectly) the calcium reabsorption from GI-tract.
*E. Decreasing the phosphate excretion.
7. Under normal physiological conditions the filtered load of which following
is higher than the Tm of reabsorption?
*A. Phosphate.
B. Lactate.
C. Calcium.
D. Citrate.
E. Glucose.
8. 60-70% of filtered load of magnesium is reabsorbed in the
A. Proximal tubule.
B. Distal tubule.
C. Collecting duct.
*D. Descending limb of Hanle’s loop.
E. Ascending limb of Hane’s loop.
Chapter 7.
1. A 40-year-old woman is admitted to the intensive care unit with
hypotension and shortness of breath. Arterial blood gases reveal: PCO2= 10
mmHg and bicarbonate, 12 mEq/L. What is her acid-base status?
A. Normal.
B. Respiratory acidosis.
C. Metabolic acidosis.
D. Metabolic alkalosis.
*E. Respiratory alkalosis.
2. Which one of the following statements about ammonia is correct?
A. It is impermeable to the epithelial cells of the proximal tubule.
B. It is classified as a titratable acid.
C. It is produced by epithelial cells in the distal nephron.
D. It induces a bicarbonate concentration in the plasma.
*E. Its synthesis is increased in respiratory acidosis.
3. A 29-year-old woman with a 3-day history of severe diarrhea comes to
the emergency room. She is hyperventilating and has postural
hypotensen. Laboratory tests show:
Serum:
[Na+] = 137 mEq/L
[Cl-] = 114 mEq/L
[HCO-3] = 13 mEq/L
Arterial blood values:
pH = 7.23
PCO2 = 26 mmHg
The most likely diagnosis is:
A. Metabolic acidosis with abnormal anionic gap + partially respiratory
compensation.
B. * Metabolic acidosis with normal anionic gap + partially respiratory
compensation.
C. Metabolic alkalosis + partially respiratory compensation.
D. Respiratory acidosis + partially metabolic compensation.
E. Respiratory alkalosis + partially metabolic compensation.
4. A 18-year-old girl is admitted to the hospital with a 4-day history of
generalized muscle cramps. Laboratory tests show:
Serum:
[Na+] = 149 mEq/L
[K+] = 2.0 mEq/L
Arterial blood:
pH = 7.50
[HCO-3] = 40 mEq/L
Which of the following is the most likely cause of this patient’s hypokalemic
alkalemia?
*A.
B.
C.
D.
E.
Hyperaldosteronism.
Hyperventilation.
Persistent diarrhea.
Renal failure.
Diabetes.
5. In which type of metabolic acidosis the ‘anionic gap’ is normal:
*A. Diarrhea.
B. Renal failure.
C. Diabetic ketoacidosis.
D. Lactic acidosis.
E. Ingestion of acidic drug.
6. Under normal physiological conditions the rate of net
nonvolatile acid (mEq/day) produced in ECF must be equal to:
A.
B.
*C.
D.
E.
Filtered load of H+.
Tubular secretion of H+.
Net acid excretion (NAE).
Tubular reabsorption of HCO3-.
Excretion rate of HCO3-.
7. A respiratory acidosis that results in an increase in the
concentration of hydrogen ions in arterial blood from 40 nM (pH
7.4) to 50 nM (pH 7.3) would
*A. Stimulate the peripheral and central chemo-receptors.
B. Decrease the amount of ammonium excreted in the urine.
C. Stimulate the osmo-receptors.
D. Increase the pH of the urine.
E. None is correct.
8. Use the following laboratory values to calculate the net acid excretion:
Plasma pH = 7.2, Urine flow = 1.2 L/day, Urine bicarbonate = 2 mEq/day, Urine
titratable acids = 24 mEq/L, Urine ammonium = 38 mEq/L, Urine pH = 5.4.
A. 55 mEq/L.
B. 64 mEq/L.
C. 68 mEq/L.
*D. 72 mEq/L.
E. 76 mEq/L.